Linear Algebra dimensions proof

Click For Summary
The discussion focuses on determining necessary and sufficient conditions for the equality dim(W_1 ∩ W_2) = dim(W_1) in the context of subspaces W1 and W2 of a finite-dimensional vector space V. It is clarified that the problem asks for conditions under which this equality holds true. A suggestion is made that if dim(W1) = m and dim(W2) = n, with n ≥ m, then W1 could be nested within W2 (W_1 ⊆ W_2). However, while this is a plausible condition, it is noted that the statement does not constitute a proof of the implication in either direction. The discussion emphasizes the need for a rigorous proof to establish the relationship between the dimensions of the subspaces.
zcd
Messages
197
Reaction score
0

Homework Statement


Let W1 and W2 be subspaces of a finite-dimensional vector space V. Determine necessary and sufficient conditions on W1 and W2 so that dim(W_1 \cap W_2)=dim(W_1)

Homework Equations


Replacement Theorem

The Attempt at a Solution


To clarify on the question: is the problem asking for conditions such that condition\iff dim(W_1 \cap W_2)=dim(W_1)?
If it is, is it possible to say that let dim(W1)=m and dim(W2)=n, n≥m and W1 is nested inside W2, so W_1 \subseteq W_2?
 
Last edited:
Physics news on Phys.org
zcd said:
To clarify on the question: is the problem asking for conditions such that condition\iff dim(W_1 \cap W_2)=dim(W_1)?

Yes.

zcd said:
If it is, is it possible to say that let dim(W1)=m and dim(W2)=n, n≥m and W1 is nested inside W2, so W_1 \subseteq W_2?
"W_1 \subseteq W_2" is a reasonable guess for what the right condition might be, but what you have written is not a proof of either direction of the implication.
 
Question: A clock's minute hand has length 4 and its hour hand has length 3. What is the distance between the tips at the moment when it is increasing most rapidly?(Putnam Exam Question) Answer: Making assumption that both the hands moves at constant angular velocities, the answer is ## \sqrt{7} .## But don't you think this assumption is somewhat doubtful and wrong?

Similar threads

  • · Replies 10 ·
Replies
10
Views
1K
Replies
34
Views
3K
  • · Replies 15 ·
Replies
15
Views
2K
Replies
15
Views
2K
  • · Replies 8 ·
Replies
8
Views
2K
  • · Replies 4 ·
Replies
4
Views
2K
  • · Replies 7 ·
Replies
7
Views
2K
  • · Replies 3 ·
Replies
3
Views
6K
Replies
1
Views
1K
  • · Replies 2 ·
Replies
2
Views
3K